LSAT and Law School Admissions Forum

Get expert LSAT preparation and law school admissions advice from PowerScore Test Preparation.

 Administrator
PowerScore Staff
  • PowerScore Staff
  • Posts: 8917
  • Joined: Feb 02, 2011
|
#59031
Please post your questions below!
 mikrnej
  • Posts: 4
  • Joined: Apr 28, 2018
|
#59482
Hi there,
I had a question with why the correct answer is E. I chose D. I recall attempting to diagram this on test day, but was limited on time and felt that the flaw was confusing the sufficient and necessary conditions in the conclusion. If all apartments are in old houses, can we conclude that most old houses will contain apartments?

Thank you!
 Brook Miscoski
PowerScore Staff
  • PowerScore Staff
  • Posts: 418
  • Joined: Sep 13, 2018
|
#59506
Mike,

Reacting to the stimulus is important for this question. When I read it, my reaction is, "why can't all the apartments be in the same great big house?" That gets me straight to answer choice E.

I realize that the stimulus starts with the word "all," but the stimulus is not based on conditional reasoning. Words like "all" can introduce conditional reasoning, but they can also introduce number concepts. In this stimulus, with "twice as many" and "most" and "more," we are dealing with some kind of numbers idea rather than conditional reasoning, so you should eliminate D immediately. Be careful not to be trapped into doing conditional diagramming before you know that the stimulus is truly based on conditional reasoning.

If you recognize that this is a numbers-based argument, you are looking for ways to violate the numbers condition. The easiest way to get a whole bunch of apartments without most houses having apartments is to concentrate the apartments in a few houses, and that's where (E) is going.
 Jbontemps
  • Posts: 3
  • Joined: Nov 04, 2018
|
#60074
I also chose D (although I can see now that only the first sentence seems to be conditional). After rereading the conclusion and prephrasing, I see that the flaw exists in there being the possibility that maybe one or just a few of the old houses contained a substantial number of the apartments on 20h Av. But can’t see how answer choice E says that...

I interpreted E as saying a number of old houses contained 3 or more apartments, but the conclusion already says most contain more than one, which includes possibly 3 or more...I didn’t take that as explicitly or even implicitly saying that only a few, not most, of the houses held a substantial number of the apartments that accounted for there being twice as many apartments as Old houses...
 Malila Robinson
PowerScore Staff
  • PowerScore Staff
  • Posts: 296
  • Joined: Feb 01, 2018
|
#60263
Hi Jbontemps,
The arguments says that there are twice as many apartments as there are old houses, so IF we distributed them evenly that would mean that every old house would have 2 apartments. There is nothing there to show that there is an even distribution though. In fact we have absolutely no idea how they are distributed. Yet the conclusion claims that we know that most houses have more than one apartment.
The flaw is that we have no idea that more than 50% of the old houses contain multiple apartments. Maybe all of the apartments are in one old house, or maybe they are split between 30% of the houses. We just don't know. Answer E is attempting to show this error by highlighting the fact that even though there are twice as many apartments we don't know they are split in a way that relates to that doubled fact.
Hope that helps!
-Malila
User avatar
 lsatmiracle
  • Posts: 6
  • Joined: Mar 17, 2021
|
#86209
i understand conceptually why E is the right answer but i'm having trouble with the wording. i didn't pick E because i thought that the argument was actually accounting for E because doesn't "most old houses having more than 1 apt" INCLUDE "significant # of old houses containing more than 3 apts"? i was confused about this. i took "significant number" as being "most" or am i just thinking too far into this?
User avatar
 JocelynL
  • Posts: 51
  • Joined: Dec 22, 2020
|
#86232
lsatmiracle wrote:i understand conceptually why E is the right answer but i'm having trouble with the wording. i didn't pick E because i thought that the argument was actually accounting for E because doesn't "most old houses having more than 1 apt" INCLUDE "significant # of old houses containing more than 3 apts"? i was confused about this. i took "significant number" as being "most" or am i just thinking too far into this?
I ruled this answer choice out for the exact same reason. I took "significant number" to also mean "most", but then I looked up what "significant number" meant in the LSAT world and came across this really good article posted by power score "A qualitative analysis" where they explain that a significant number just means >0%. Hope this helps! Lets not get burned by this type of question again! :)

https://blog.powerscore.com/lsat/a-qual ... -the-lsat/
User avatar
 Ryan Twomey
PowerScore Staff
  • PowerScore Staff
  • Posts: 141
  • Joined: Mar 04, 2021
|
#86234
Yes I would say significant number means the same as many or often, which I define as happening in at least two instances.

I consider some to be a statement of its own meaning at least one instance, which becomes very important in assumption necessary questions when you negate some it becomes none.

Hope this helps.
User avatar
 Overthinker99
  • Posts: 9
  • Joined: May 30, 2023
|
#105865
I understand that sufficiency and necessity are concepts we generally relegate to conditional logic, however, their idea is not exclusive to formal conditional logic. Technically, saying most houses would have more than one apartment includes the possibility that they all have two. This is one of many possible explanations that would be sufficient to explain the phenomena in the stimulus. So the error can therefore be described as confusing an explanation that would be sufficient for one that is therefore necessary--which led the stimulus to make the false conclusion that this explanation is necessarily true. I do not understand why this is wrong.
User avatar
 Chandler H
PowerScore Staff
  • PowerScore Staff
  • Posts: 54
  • Joined: Feb 09, 2024
|
#105886
Overthinker99 wrote: Wed Apr 03, 2024 12:17 am I understand that sufficiency and necessity are concepts we generally relegate to conditional logic, however, their idea is not exclusive to formal conditional logic. Technically, saying most houses would have more than one apartment includes the possibility that they all have two. This is one of many possible explanations that would be sufficient to explain the phenomena in the stimulus. So the error can therefore be described as confusing an explanation that would be sufficient for one that is therefore necessary--which led the stimulus to make the false conclusion that this explanation is necessarily true. I do not understand why this is wrong.
Hi Overthinker99,

I think you are confused as to the definitions of "sufficient" and "necessary." A sufficient condition is not simply an explanation that is sufficient to explain the conclusion. A sufficient condition is a proposition whose truth assures the truth of another proposition, so it's actually the opposite of what you're saying.

Also, answer choice (D) claims that the sufficient and necessary conditions are confused. What would be the confusion within the stimulus? There isn't really anything we can directly point to.

Does this make sense?

Get the most out of your LSAT Prep Plus subscription.

Analyze and track your performance with our Testing and Analytics Package.